LSAT and Law School Admissions Forum

Get expert LSAT preparation and law school admissions advice from PowerScore Test Preparation.

User avatar
 Dave Killoran
PowerScore Staff
  • PowerScore Staff
  • Posts: 5850
  • Joined: Mar 25, 2011
|
#44098
Complete Question Explanation
(The complete setup for this game can be found here: lsat/viewtopic.php?t=8548)

The correct answer choice is (A)

Per the final inference, when G is in the forest, then S must be in the forest, and answer choice (A) is proven correct. A review of the entire inference chain shows that, when G is in the forest, then H is not in the forest (from rule #1), and when H is not in the forest, then neither J or M is in the forest (from rule #2). When J is not in the forest, then S must be in the forest (from rule #4).

In this question, be sure to avoid making a Mistaken Reversal: although from the third rule we know that when W is in the forest then G must be in the forest, this does not mean that when G is in the forest that W must also be in the forest.

Get the most out of your LSAT Prep Plus subscription.

Analyze and track your performance with our Testing and Analytics Package.